SlideShare ist ein Scribd-Unternehmen logo
1 von 47
Downloaden Sie, um offline zu lesen
1
                                                                 6.       sec 2t tan 2t dt =       sec 2t + c
                                                                                                 2

                                                                 7.       (x2 + 4)2 dx =       (x4 + 8x2 + 16)dx
                                                                          x5  8
                                                                      =      + x3 + 16x + c
                                                                          5   3
Chapter 6
                                                                 8.       x(x2 + 4)2 dx =         (x5 + 8x3 + 16x)dx
                                                                          x6
                                                                      =      + 2x4 + 8x2 + c
Integration                                                              3
                                                                          6
                                                                                  3     x
                                                                 9.           dx = tan−1 + c
Techniques                                                            16 + x2
                                                                         2
                                                                                  4
                                                                                   1
                                                                                        4

                                                                10.          2
                                                                               dx = tan−1 x + c
                                                                      4 + 4x       2
                                                                                  1
                                                                11.       √               dx
                                                                              3 − 2x − x2
6.1       Review of Formulas                                                         1                          x+1
                                                                      =                          dx = arcsin           +c
          and Techniques                                                        4 − (x +   1)2                   2

                     1 ax                                                  x+1
 1.       eax dx =     e + c, for a = 0.                        12.       √          dx
                     a                                                   3 − 2x − x2
                                                                         1      −2(x + 1)
                                                                      =−                      dx
                           1                                             2      4 − (x + 1)2
 2.       cos(ax)dx =        sin(ax) + c, for a = 0.
                           a                                             1
                                                                      = − · 2[4 − (x + 1)2 ]1/2 + C
                                                                         2
                1                     1          1
 3.       √           dx =                           dx               = − 4 − (x + 1)2 + c
              a2 − x2                      x 2   a
                                     1−    a                                4
                x         1                                     13.                dx
      Let u =     , du = dx.                                           5 + 2x + x2
                a         a                                                     1                               x+1
                1                                                     =4                dx = 2 tan−1                     +c
      =     √          du = sin−1 (u) + c                                  4 + (x + 1)2                          2
              1 − u2
           −1 x
      = sin          + c, a > 0.                                          4x + 4
                 a                                              14.                dx
                                                                       5 + 2x + x2
               b                                                             2(x + 1)
 4.          √        dx                                              =2                dx = 2 ln | 4 + (x + 1)2 | + c
          |x| x2 − a2                                                      4 + (x + 1)2
                  b              1                                          4t
      =                               dx                        15.                 dt
                     x 2         a                                      5 + 2t + t2
              |x|    a −1
                                                                             4t + 4                       4
               x        1                                             =                dt −                      dt
      Let u = , du = dx and |au| = |x| .                                  5 + 2t + t2                5 + 2t + t2
               a        a                                                                                    t+1
                  b                                                   = 2 ln 4 + (t + 1)
                                                                                           2
                                                                                                 − 2tan−1           +c
      =         √         du                                                                                   2
            |au| u2 − 1
         b           1                                                    t+1             2 (t + 1)
      =            √        du                                  16.                dt =             dt
        |a|    |u| u2 − 1                                              t2 + 2t + 4              2
                                                                                        (t + 1) + 3
         b                                                             1
      =     sec−1 (u) + c                                                          2
                                                                      = ln (t + 1) + 3 + c
        |a|                                                            2
           b        x
      =       sec−1   + c, a > 0.                               17.
                                                                                      1
                                                                          e3−2x dx = − e3−2x + c
          |a|       a                                                                 2
                       1                                                              3
 5.       sin(6t)dt = − cos(6t) + c                             18.       3e−6x dx = − e−6x + c
                       6                                                              6

                                                          360
6.1. REVIEW OF FORMULAS AND TECHNIQUES                                                                                                                  361

                                 2 −1/3                          30. Let u = ex , du = ex dx
19. Let u = 1 + x2/3 , du =        x    dx
                                 3                                       ex                 1
                   4                 3                               √         dx =     √        du
                            dx = 4        u−1 du                       1 − e2x            1 − u2
           x1/3 (1 + x2/3 )          2
                                                                       = sin−1 u + C = sin−1 ex + c
                                                     2/3
      = 6 ln |u| + C = 6 ln |1 + x                         |+c
                                                                 31. Let u = x2 , du = 2xdx
                             3                                              x          1       1
20. Let u = 1 + x3/4 , du = x−1/4 dx                                    √        dx =      √        du
                             4                                            1 − x4       2     1 − u2
           2                     2
                dx =                      dx                           1                 1
       x1/4 + x          x1/4 (1 + x3/4 )                            = sin−1 u + C = sin−1 x2 + c
          4                8                                           2                 2
    =2          u−1 du = ln |u| + C
          3                3                                     32. Let u = 1 − x4 , du = −4x3 dx
      8                                                                   2x3            1
    = ln |1 + x3/4 | + c                                                √        dx = −     u−1/2 du
      3                                                                   1−x  4         2
            √         1                                                = −u1/2 + C = −(1 − x4 )1/2 + c
21. Let u = x, du = √ dx
          √          2 x
       sin x                                                              1+x
         √ dx = 2 sin udu                                        33.             dx
          x                                                               1 + x2
                           √                                                   1        1     2x
    = −2 cos u + C = −2 cos x + c                                      =           dx +             dx
                                                                            1 + x2      2  1 + x2
               1          1                                                         1
22. Let u =      , du = − 2 dx                                         = tan−1 x + ln |1 + x2 | + c
               x         x                                                          2
        cos(1/x)
                   dx = − cos udu                                                    1
            x2                                                   34.        √           dx
                             1                                                      x+x
      = − sin u + C = − sin + c
                             x                                                                    1
                                                                       =            x−1/2 ·             dx
23. Let u = sin x, du = cos xdx                                                                1 + x1/2
           π                             0
                                                                       = 2 ln | 1 + x1/2 | + c
               cos xesin x dx =              eu du = 0
       0                             0
                                                                            ln x2                                1
                                             2                   35.              dx = 2                 ln x                 dx
24. Let u = tan x, du = sec xdx                                               x                                  x
           π/2                                   1                                                  1
                   sec2 xetan x dx =                 eu du             Let u = ln x, du =             dx.
                                                                                                    x
       0                                     0
                                                                                                              2
               u
                   1
                                                                       =2               u du = u2 + c = (ln x) + c
      =e               =e−1
                   0
                                                                            3                            3                         3
           0                                                                                                                 x3            26
25.                sec x tan xdx                                 36.            e2 ln x dx =                 x2 dx =                   =
       −π/4                                                             1                            1                       3     1        3
                       0             √                                      4
      = sec x                  =1−       2                                       √
                       −π/4                                      37.            x x − 3dx
                                                                        3
           π/2                                                                      4              √
                                                 π/2
26.                csc2 xdx = − cot x                   =1             =                (x − 3 + 3) x − 3dx
       π/4                                       π/4                            3
                                                                                    4                                    4
                           3         2                                 =                (x − 3)3/2 dx + 3                    (x − 3)1/2 dx
27. Let u = x , du = 3x dx                                                      3                                    3
      x2         1      1                                                   2
                                                                                                 4
                                                                                                        2
                                                                                                                                           4
                                                                                                                                                   12
           dx =             du                                         =      (x − 3)5/2           + 3 · (x − 3)3/2                            =
    1 + x6       3   1 + u2                                                 5                           3                                           5
                                                                                                 3                                         3
      1                1
    = tan−1 u + C = tan−1 x3 + c                                            1
      3                3                                         38.            x(x − 3)2 dx
                                                                        0
             x5       1                                                             1
28.               dx = ln(1 + x6 ) + c                                 =                (x3 − 6x2 + 9x)dx
           1 + x6     6
                                                                                0
                                                                                                                 1
         1              x                                                           x4        9                               11
29. √         dx = sin−1 + c                                           =               − 2x3 + x2                    =
        4−x 2           2                                                           4         2                  0             4
362                                                                       CHAPTER 6. INTEGRATION TECHNIQUES

            4                                                                      2
                x2 + 1
 39.             √ dx                                                   47.            f (x)dx
        1          x                                                           0
                    4                    4                                                 1                    2
                                                                                                  x                   x2
       =                x3/2 dx +            x−1/2 dx                         =                  2+1
                                                                                                     dx +                  dx
                1                    1                                                 0       x            1       x2 + 1
                                                                                              1      2
                            4                                                 1                                  1
            2 5/2                            4       72                     = ln |x2 + 1| +              1− 2         dx
       =      x                 + 2x1/2          =                            2               0    1         x +1
            5               1                1        5                       1                           2
                                                                            = ln 2 + (x − arctan x)
            0                                        0
                                                                              2                           1
                            2            1 −x2                e−4 − 1         ln 2         π
 40.            xe−x dx = −                e              =                 =      + 1 + − arctan 2
        −2                               2           −2          2              2          4
                                                                                   4x + 1
               5         5       x                                      48.                     dx
 41.               dx = √ arctan √ + c                                         2x2 + 4x + 10
            3 + x2        3       3
               5                                                                       4x + 4                       3
                   dx: N/A                                                  =        2 + 4x + 10
                                                                                                  dx −         2 + 4x + 10
                                                                                                                           dx
            3 + x3                                                                2x                        2x
                                                                                                       3           1
                                                                            = ln |2x2 + 4x + 10| −                       dx
                       1                                                                               2    (x + 1)2 + 4
 42.        sin(3x)dx =    sin(3x)3dx
                       3                                                                                            3           x+1
       Let u = 3x, du = 3dx.                                                  = ln |2x2 + 4x + 10| −                  tan−1           +c
         1                  1                                                                                       4            2
       =      (sin u)du = − cos u + c
         3                  3                                                       1
           1                                                            49.                dx = tan−1 (x) + c.
       = − cos(3x) + c.                                                         (1 + x2 )
           3                                                                        x            1       2x
                                                                                        2)
                                                                                           dx =                 dx
                                                                                (1 + x           2    (1 + x2 )
            sin3 xdx =              (sin2 x) sin xdx                            1
                                                                              = ln 1 + x2 + c.
                                                                                2
       =            (1 − cos2 x)sin xdx                                            x2              x2 + 1 − 1
                                                                                           dx =                 dx
       Let u = cos x, du = − sin xdx.                                           (1 + x2 )            (1 + x2 )
                                                                                      2
       =                1 − u2 (−du) =                u2 du −     du                x +1                  1
                                                                              =       2 + 1)
                                                                                             dx −                dx
                                                                                   (x                 (1 + x2 )
            u3     cos3 x                                                                       1
       =       −u=        − cos x.                                            = dx −                  dx
            3        3                                                                      (1 + x2 )
                                                                              = x − tan−1 (x) + c.
 43.        ln xdx: N/A                                                             x3          1       x2
                                                                                        2)
                                                                                           dx =               2xdx
       Substituting u = ln x,                                                    (1 + x         2   (1 + x2 )
         ln x      1
              dx = ln2 x + c                                                  Let u = x2 , du = 2xdx.
          2x       4                                                            1       u         1    u+1−1
                                                                              =             du =                du
                                                                                2     1+u         2      1+u
 44. Substituting u = x4                                                        1       u+1             1
                                                                              =               du −         du
         x3         1                                                           2       1+u           1+u
              dx = arctan x4 + c
       1 + x8       4                                                           1                 1
                                                                              =        du −           du
         x4                                                                     2               1+u
              dx: N/A                                                           1
       1 + x8                                                                 = (u − ln (1 + u)) + c
                                                                                2
                                                                                1       1
 45.        e−x dx: N/A
                        2
                                                                              = x2 − ln 1 + x2 + c.
                                                                                2       2
       Substituting u = −x2                                                   Hence we can generalize this as follows,
              2        1   2                                                         xn
         xe−x dx = − e−x + c                                                       1 + x2
                                                                                            dx
                       2
                                                                                    1                            xn−2
                                                                              =        xn−1 −                          dx
 46.        sec xdx: N/A                                                           n−1                          1 + x2
                                                                                     x         1                   1
            sec2 xdx = tan x + c                                        50.               dx =                         2xdx
                                                                                   1 + x4      2                1 + x4
6.2. INTEGRATION BY PARTS                                                                               363

      Let u = x2 , du = 2xdx.                                          1 2x               1 2x
                                                                xe2x dx =xe −               e dx
        1         1           1                                        2                  2
      =                du = tan−1 (u) + c                      1      1
        2     1 + u2          2                               = xe2x − e2x + c.
        1                                                      2      4
      = tan−1 x2 + c.
        2                                                  4. Let u = ln x, dv = x dx
           x3            1        1                                 1              x2
               4
                  dx =                4x3 dx                  du = dx and v =         .
         1+x             4     1 + x4                               x              2
      Let u = 1 + x4 , du = 4x3 .                                            1              1
                                                                x ln x dx = x2 ln x −         x dx
        1     1         1                                                    2              2
      =         du = ln (u) + c                                 1            1
        4     u         4                                     = x2 ln x − x2 + c.
        1                                                       2            4
      = ln 1 + x4 + c.
        4                                                  5. Let u = ln x, dv = x2 dx
           x5                                                       1          1
                  dx                                          du = dx, v = x3 .
         1 + x4                                                     x          3
        1        x4                                               2          1 3             1 3 1
      =                2xdx                                     x ln xdx = x ln x −            x · dx
        2     1 + x4                                                         3               3    x
      Let u = x2 , du = 2xdx.                                   1 3         1
                                                              = x ln x −         x2 dx
        1        u2           1     u2 + 1 − 1                  3           3
      =                du =                    du               1           1
        2     1+u    2        2       1 + u2                  = x3 ln x − x3 + c.
        1        u2 + 1               1                         3           9
      =                  du −             du
        2        1 + u2            1 + u2                                       1
                                                           6. Let u = ln x, du =  dx.
        1                     1                                                 x
      =         du −               du                            ln x              u2      1
        2                  1 + u2                                     dx =   udu =    + c = (ln x)2 + c.
        1                                                         x                2       2
      =     u − tan−1 (u) + c
        2
        1 2                                                7. Let u = x2 , dv = e−3x dx
      =     x − tan−1 x2 + c.                                                    1
        2                                                     du = 2xdx, v = − e−3x
      Hence we can generalize this as follows,                                   3
         x4n+1          1 x2n−2             x4(n−1)+1         I = x2 e−3x dx
                 dx =                  −              dx
         1 + x4         2 n−1                1 + x4               1                1
      and                                                     = − x2 e−3x −      − e−3x · 2xdx
                                                                  3                3
         x4n+3      1      x2n         x4(n−1)+3                  1          2
               dx =              −               dx           = − x2 e−3x +      xe−3x dx
        1+x  4      4       n           1 + x4                    3          3
                                                              Let u = x, dv = e−3x dx
                                                                             1
6.2      Integration by Parts                                 du = dx, v = − e−3x
                                                                             3
                                                                    1
                                                              I = − x2 e−3x
 1. Let u = x, dv = cos xdx                                         3
    du = dx, v = sin x.                                             2    1                1
                                                                  +    − xe−3x −       − e−3x dx
        x cos xdx = x sin x −      sin xdx                          3    3                3
                                                                  1          2          2
      = x sin x + cos x + c                                   = − x2 e−3x − xe−3x +           e−3x dx
                                                                  3          9          9
                                                                  1          2           2 −3x
 2. Let u = x, dv = sin 4xdx                                  = − x2 e−3x − xe−3x −         e    +c
                   1                                              3          9          27
    du = dx, v = − cos 4x
                   4                                       8. Let u = x3 , du = 3x2 dx.
                                                                     3       1          1
      x sin 4x dx                                               x2 ex dx =       eu dx = eu + c
                                                                             3          3
         1                1                                     1 x3
      = − x cos 4x − − cos 4x dx                              = e + c.
         4                4                                     3
         1            1
      = − x cos 4x +    sin 4x + c.
         4           16                                    9. Let I =    ex sin 4xdx
 3. Let u = x, dv = e2x dx                                    u = ex , dv = sin 4xdx
                  1                                                              1
    du = dx, v = e2x .                                        du = ex dx, v = − cos 4x
                  2                                                              4
364                                                            CHAPTER 6. INTEGRATION TECHNIQUES

             1                   1                                                   1
      I = − ex cos 4x −        − cos 4x ex dx                     du = cos xdx v = − cos 2x
             4                   4                                                   2
             1 x          1                                           1              1   1
        = − e cos 4x +          x
                               e cos 4xdx                         I = cos x sin 2x +   − cos 2x sin x
             4            4                                           2              2   2
                                                                             1
      Use integration by parts again, this time let                 −      − cos 2x cos xdx
      u = ex , dv = cos 4xdx                                                 2
                       1                                                  1               1              1
      du = ex dx, v = sin 4x                                          =     cos x sin 2x − cos 2x sin x + Idx
                       4                                                  2               4              4
             1 x
      I = − e cos 4x                                               So,
             4                                                    3      1             1
             1 1 x               1                                  I = cos x sin 2x − cos 2x sin x + c1
           +      e sin 4x −       (sin 4x)ex dx                  4      2             4
             4 4                 4
             1             1               1                           2             1
      I = − ex cos 4x + ex sin 4x − I                             I = cos x sin 2x − cos 2x sin x + c
             4            16              16                           3             3
       So,
      17        1             1                               12. Here we use the trigonometric identity:
         I = − ex cos 4x + ex sin 4x + c1                         sin 2x = 2 sin x cos x.
      16        4            16
             4              1                                     We then make the substitution
      I = − ex cos 4x + ex sin 4x + c
             17             17                                    u = sin x, du = cos x dx.
 10. Let, u = e2x , dv = cos x dx so that,                            sin x sin 2x dx =      2 sin2 x cos x dx
     du = 2e2x dx and v = sin x.
       e2x cos x dx                                                               2 3       2
                                                                  =       2u2 du =  u + c = sin3 x + c
                                                                                  3         3
      = e2x sin x − 2         e2x sin x dx                        This integral can also be done by parts, twice.
                                                                  If this is done, an equivalent answer is ob-
      Let, u = e2x , dv = sin x dx so that,                       tained:
      du = 2e2x dx and v = − cos x.                               1                2
                                                                    cos x sin 2x − cos 2x sin x + c
          e2x sin x dx                                            3                3
                                                              13. Let u = x, dv = sec2 xdx
             2x                     2x
      = −e        cos x + 2     e        cos x dx                 du = dx, v = tan x

          e2x cos x dx                                                x sec2 xdx = x tan x −        tan xdx
                                                                                  sin x
      = e2x sin x + 2e2x cos x − 4             e2x cos x dx       = x tan x −           dx
                                                                                  cos x
      Now we notice that the integral on both of                  Let u = cos x, du = − sin xdx
                                                                                              1
      these is the same, so we bring them to one side               x sec2 xdx = x tan x +      du
      of the equation.                                                                        u
                                                                  = x tan x + ln |u| + c
      5    e2x cos x dx                                           = x tan x + ln |cos x| + c

      = e2x sin x + 2e2x cos x + c1                           14. Let u = (ln x)2 , dv = dx
                                                                         ln x
          e2x cos x dx                                            du = 2      dx, v = x
                                                                          x
          1 2x       2
      =     e sin x + e2x cos x + c                               I = (ln x)2 dx
          5          5
                                                                                             ln x
                                                                      = x(ln x)2 −     x·2        dx
 11. Let I =         cos x cos 2xdx                                                           x
      and u = cos x, dv = cos 2xdx                                    = x(ln x)2 − 2      ln xdx
                         1
      du = sin xdx, v = sin 2x                                    Integration by parts again,
                         2                                                               1
          1                   1                                   u = ln x, dv = dxdu = dx, v = x
      I = cos x sin 2x −        sin 2x(− sin x)dx                                        x
          2                   2                                                               1
                                                                             2
          1                1                                      I = x(ln x) − 2 x ln x − x · dx
        = cos x sin 2x +        sin x sin 2xdx                                                x
          2                2
      Let,u = sin x, dv = sin 2xdx                                = x(ln x)2 − 2x ln x + 2         dx
6.2. INTEGRATION BY PARTS                                                                                                 365

    = x(ln x)2 − 2x ln x + 2x + c                  20. Let u = 2x, dv = cos x dx
                                                       duπ= 2 dxandv = sin x.                               π
                              2
15. Let u = x2 , dv = xex dx so that, du = 2x dx                  2x cos xdx = 2x sin x|0 − 2
                                                                                               π
                                                                                                                sin xdx
                1 2                                       0                                             0
    and v = ex (v is obtained using substitu-            = (2x sin x +
                                                                                       π
                                                                              2 cos x)|0   = −4.
                2
    tion).
            2       1   2        2                            1
       x3 ex dx = x2 ex − xex dx                   21.            x2 cos πxdx
                    2
                                                          0
       1      2   1 2
    = x2 ex − ex + c                                     Let u = x2 , dv = cos πxdx,
       2          2                                                          sin πx
                                                         du = 2xdx, v =             .
                                                                                π
                                  x                         1                              1       1
16. Let u = x2 , dv =                 dx                                          sin πx             sin πx
                                 3/2                          x2 cos πxdx = x2               −              2xdx
                        (4 + x2 )                         0                           π 0        0      π
                           1                                             2 1
    du = 2xdx, v = − √                                   = (0 − 0) −           x sin (πx) dx
                         4 + x2                                          π 0
              3                                                     1
            x                         x                        2
                3/2
                    dx = x2               3/2
                                              dx         =−           x sin (πx) dx
       (4 + x2 )                 (4 + x2 )                     π 0
            x2              1                            Let u = x, dv = sin(πx)dx,
    = −√           +    √        2xdx                                       cos(πx)
           4+x   2        4 + x2                         du = dx, v = −               .
             x2                                                                 π
                                                                  1
    =−               + 2 (4 + x2 ) + c.                     2
           (4 + x2 )                                     −          xsin(πx)dx
                                                            π 0
                                                                                  1       1
17. Let u = ln(sin x), dv = cos xdx                            2      x cos(πx)                cos(πx)
                                                         =−         −               −        −           dx
            1                                                  π           π      0     0          π
    du =        · cos xdx, v = sin x                                                                             1
          sin x                                             2            cos π        1 sin(πx)
                                                         =−            (−      − 0) +
    I = cos x ln(sin x)dx                                   π              π          π     π                    0
    = sin x ln(sin x)                                       2          1   1              2
                     1                                   =−              + (0 − 0) = − 2
      − sin x ·          · cos xdx                          π          π π               π
                   sin x
                                                              1
    = sin x ln(sin x) −       cos xdx
                                                   22.            x2 e3x dx
    = sin x ln(sin x) − sin x + c                         0
                                                         Let u = x2 , dv = e3x dx,
                                                                            e3x
18. This is a substitution u = x2 .                      du = 2xdx, v =           .
                    1                                                         3
      x sin x2 dx =      sin udu                            1
                                                                           x2 e3x
                                                                                    1       1 3x
                                                                                             e
                    2                                         x2 e3x dx =             −          2xdx
        1                1                                                    3 0             3
    = − cos u + c = − cos x2 + c.                         0                               0
        2                2                                  1 3             2 1 3x
                                                         =      e −0 −              xe dx.
19. Let u = x, dv = sin 2xdx                                3               3 0
                  1                                      Let u = x, dv = e3x dx,
    du = dx, v = − cos 2x                                                e3x
                  2                                      dv = dx, v =        .
         1
                                                                          3
             x sin 2xdx                                  e3     2 1 3x
     0                                                       −         xe dx
       1
                          1
                          1
                              1                           3     3 0
    = − x cos 2x −          − cos 2x dx                     e3     2      e3x
                                                                                1       1 3x
                                                                                         e
       2          0     0     2                          =      −      x          −          dx
       1                      1 1                            3     3       3 0        0   3
    = − (1 cos 2 − 0 cos 0) +     cos 2xdx                                                 1
       2                      2 0                          e3    2            e3               e3x
                              1                          =     −                   −               dx
       1         1 1                                       3     3            3        0        3
    = − cos 2 +        sin 2x                                3                                     1
       2         2 2          0                            e     2            e3       e3x
       1         1                                       =     −                   −
    = − cos 2 + (sin 2 − sin 0)                            3     3             3        9          0
       2         4                                         e 3
                                                                 2            e3     1 3
       1         1                                       =     −                   −   e −1
    = − cos 2 + sin 2                                      3     3            3      9
       2         4
366                                                                CHAPTER 6. INTEGRATION TECHNIQUES

         e3   2e3    2 3                                                      x cos (ax) sin (ax)
       =    −     +     e −1                                            =−              +         + c, a = 0.
         3     9    27                                                            a         a2
         e3   2e3   2e3    2   5e3   2
       =    −     +     −    =     −
         3     9    27    27   27    27                           27.       (xn ) (ln x) dx =    (ln x) (xn ) dx
            10
                                                                        Let u = ln x, dv = xn dx,
 23.             ln 2xdx                                                      1          xn+1
        1                                                               du = dx, v =            .
       Let u = ln 2x, dv = dx                                                 x         (n + 1)
             1
       du = dx, v = x.                                                      (ln x)(xn ) dx
             x
            10                                        10
                                         10                 1                     xn+1          xn+1 dx
                 ln (2x)dx = x ln (2x)|1 −                 x dx         = (ln x)         −
        1                                         1         x                    (n + 1)       (n + 1) x
                                        10
       = (10 ln(20) − ln 2) −                dx                           xn+1 (ln x)          xn
                                                                        =              −            dx
                                    1
                                        10
                                                                            (n + 1)         (n + 1)
       = (10 ln(20) − ln 2) − [x]1                                        x n+1
                                                                                (ln x)     x n+1
       = (10 ln(20) − ln 2) − (10 − 1)                                  =              −         2 + c, n = −1.
                                                                            (n + 1)      (n + 1)
       = (10 ln(20) − ln 2) − 9.

 24. Let, u = ln x, dv = x dx
                                                                  28.       (sin ax) (cos bx) dx
             1        x2
     du = dx, v =        .
             x         2                                                Let u = sin ax, dv = (cos bx) dx
        2                    2      2                                                           sin bx
                     1                1                                 du = a (cos ax) dx, v =        .
          x ln xdx = x2 ln x −          xdx                                                        b
      1              2       1     1 2
                           2
           1 2       1                   3                                sin ax cos bx dx
     =       x ln x − x2     = 2 ln 2 − .
           2         4     1             4                                           sin bx            sin bx
                                                                        = (sin ax)           − a                  (cos ax) dx
                                                                                        b                 b
 25.        x2 eax dx                                                      (sin ax) (sin bx) a
                                                                        =                     −        (cos ax) (sin bx) dx
                                                                                   b             b
       Let u = x2 , dv = eax dx,                                        Let u = cos ax, dv = sin bxdx,
                         eax                                                                            cos bx
       du = 2xdx, v =         .                                         du = −a (sin ax) dx, v = −              .
                          aax                                                                               b
                         e           eax
          x2 eax dx = x2        −         2xdx                          sin ax sin bx a
                           a           a                                               −        cos ax sin bx dx
                                                                              b            b
          x2 eax    2                                                      sin ax sin bx a               − cos bx
       =          −      xeax dx.                                       =                 −      cos ax
            a       a                                                            b           b                b
       Let u = x, dv = eax dx,
                      eax                                                                    − cos bx
       dv = dx, v =       .                                                            −                (− sin ax) adx
                       a                                                                          b
        2 ax
       x e        2                                                        sin ax sin bx a − cos ax cos bx
               −      xeax dx                                           =                 −
         a        a                                                              b           b            b
          x2 eax    2     eax         eax                                                 a
       =          −     x       −         dx                                           −       cos bx sin ax dx
            a       a       a          a                                                  b
          x2 eax    2 xeax         eax                                     sin ax sin bx a cos ax cos bx
       =          −             − 2 +c                                  =                 +
            a       a     a        a                                             b                  b2
                                                                                              a 2
          x2 eax    2xeax       2eax                                                      +           sin ax cos bx dx
       =          −         + 3 + c, a = 0.                                                   b
            a         a2         a
                                                                            sin ax cos bx dx
 26.        x sin (ax) dx                                                 sin ax sin bx a cos ax cos bx
                                                                        =              +
       Let u = x, dv = sin axdx,                                                b                b2
                      cos ax                                                               a 2
       du = dx, v = −        .                                                         +           sin ax cos bx dx
                         a                                                                 b
                                                                                               a   2
         xsin (ax) dx                                                     sin ax cos bx dx −            sin ax cos bx dx
                                                                                               b
             − cos (ax)            cos (ax)                               sin ax sin bx a cos ax cos bx
       =x               −      −            dx                          =              +
                 a                     a                                        b                b2
6.2. INTEGRATION BY PARTS                                                                                            367

          a2
        1−        sin ax cos bx dx                                  + (n − 1)        sinn xdx
          b2
      sin ax sin bx a cos ax cos bx                            n        sinn xdx
    =               +
             b               b2
                                                               = − sinn−1 x cos x
      sin ax cos bx dx
                                                                    − (n − 1)        sinn−2 xdx
             b2        sin ax sin bx a cos ax cos bx
    =                               +                                                 1
          b2 − a2            b              b2                      sinn xdx = −        sinn−1 x cos x
                                                                                      n
        sin ax cos bx dx                                                 n−1
                                                                    −              sinn−2 xdx
            1                                                             n
    =     2 − a2
                 (b sin ax sin bx + a cos ax cos bx) ,
        b
    a = 0 b = 0.                                         31.        x3 ex dx = ex (x3 − 3x2 + 6x − 6) + c
29. Letu = cosn−1 x, dv = cos xdx
    du = (n − 1)(cosn−2 x)(− sin x)dx, v = sin x         32.        cos5 xdx
        cosn xdx                                                1               4
                                                               =  cos4 sin x +       cos3 xdx
    = sin x cos n−1
                x                                               5               5
                                                                1
      − (sin x)(n − 1)(cosn−2 x)(− sin x)dx                    = cos4 sin x
                                                                5
    = sin x cosn−1 x                                              4 1                   2
                                                                +        cos2 x sin x +      cos xdx
                                                                  5 3                   3
      + (n − 1)(cosn−2 x)(sin2 x)dx                             1                4
                                                               = cos4 sin x +      cos2 x sin x
    = sin x cosn−1 x                                            5               15
                                                                   8
      + (n − 1)(cosn−2 x)(1 − cos2 x)dx                         +    sin x + c
                                                                  15
    = sin x cosn−1 x
      + (n − 1)(cosn−2 x − cosn x)dx                     33.        cos3 xdx
                                                                1                       2
    Thus,      cosn xdx                                        =  cos2 x sin x +             cos xdx
                                                                3                       3
                                                                1                       2
    = sin x cosn−1 x +       (n − 1) cosn−2 xdx                = cos2 x sin x +           sin x + c
                                                                3                       3
        − (n − 1)     cosn xdx.                          34.        sin4 xdx
    n     cosn xdx = sin x cosn−1 x                               1
                                                               = − sin3 x cos x +
                                                                                          3
                                                                                                sin2 xdx
                                                                  4                       4
        + (n − 1)     cosn−2 xdx                                  1                       3     1    1
                                                               = − sin3 x cos x +                 x − sin 2x
                                                                  4                       4     2    4
        cosn xdx
                                                                    1
     1                 n−1                               35.            x4 ex dx
    = sin x cosn−1 x +                cos   n−2
                                                  xdx           0
     n                  n                                                                                  1
                                                               = ex (x4 − 4x3 + 12x2 − 24x + 24)           0
30. Let u = sinn−1 x, dv = sin x dx                            = 9e − 24
    du = (n − 1) sinn−2 x cos x, v = − cos x.
        sinn xdx                                         36. Using the work done in Exercise 34,
                                                                    π/2

    = − sinn−1 x cos x                                                    sin4 xdx
                                                                0
        + (n − 1)     cos2 x sinn−2 xdx                             1             3     3
                                                                                                               π/2
                                                               =  − sin3 x cos x + x −    sin 2x
    = − sinn−1 x cos x                                              4             8    16                      0
                                                                 3π
        + (n − 1)     (1 − sin2 x) sinn−2 xdx                  =
                                                                 16
    = − sinn−1 x cos x                                              π/2
        − (n − 1)     sinn−2 xdx                         37.              sin5 xdx
                                                                0
368                                                     CHAPTER 6. INTEGRATION TECHNIQUES

                            π/2                                 π/2
           1                  4 π/2 3
      = − sin4 x cos x     +        sin xdx                           cosm xdx
           5           0      5 0                           0
           1
                       π/2                                      (n − 1)(n − 3)(n − 5) · · · 2
      = − sin4 x cos x                                     =                                  .
           5                                                       n(n − 2)(n − 4) · · · 3
                       0
                                        π/2
           4    1               2                      41. Let u = cos−1 x, dv = dx
        +     − sin2 x cos x − cos x                                  1
           5    3               3       0                  du = − √        dx, v = x
      (Using Exercise 30)                                           1 − x2
           1       π      π                                            cos−1 xdx
      =−     sin4     cos − sin4 0 cos 0                   I=
           5       2      2
           4    1    2 π       π 2       π                                                          1
        +     − sin        cos − cos                           = x cos−1 x −               x −√            dx
           5    3       2      2  3      2                                                        1 − x2
         8                                                                        x
      =                                                        = x cos−1 x +            dx  √
        15                                                                      1 − x2
                                                            Substituting u = 1 − x2 , du = −2xdx
 38. Here we will again use the work we did in Ex-
                                                                               1       1
     ercise 34.                                            I = x cos−1 x +    √     − du
                                                                                u      2
        sin6 xdx                                                          1
                                                             = x cos1 x −     u−1/2 du
          1               5                                               2
      = − sin5 x cos x +       sin4 xdx                                     1
          6               6                                  = x cos−1 x − · 2u1/2 + c
          1                                                                 2
      = − sin5 x cos x                                       = x cos−1 x − 1 − x2 + c
          6
          5    1                 3      3
        +    − sin3 x cos x + x −         sin 2x + c   42. Let u = tan−1 x, dv = dx
          6    4                 8     16                           1
          1               5                                du =         dx, v = x
      = − sin5 x cos x −     sin3 x cos x                        1 + x2
          6               24                                                                                 x
          15
        + x−
                 15
                    sin 2x + c                             I=          tan−1 xdx = x tan−1 x −                    dx
          48     96                                                                                        1 + x2
                                                            Substituting u = 1 + x2 ,
      We now just have to plug in the endpoints:                           1
           π/2                                             I = x tan−1 x − ln(1 + x2 ) + c.
                 sin6 xdx                                                  2
       0                                                                    √          1
            1              5                           43. Substituting u = x, du = √ dx
      =   − sin5 x cos x −    sin3 x cos x                                            2 x
            6              24                                        √
                            π/2                            I = sin xdx = 2 u sin udu
          15    15
        + x−        sin 2x
          48    96          0                               = 2(−u cos u + sin u) + c
                                                                   √     √         √
        15π                                                 = 2(− x cos x + sin x) + c
      =
         96                                                                 √
                                                       44. Substituting w = x
 39. m even :                                                      1        1
           π/2                                             dw = √ dx =        dx
                 sinm xdx                                        2 x       2w
                                                                           √
                                                                               x
       0
        (m − 1)(m − 3) . . . 1 π                           I=          e           dx =     2wew dx
      =                       ·                            Next, using integration by parts
          m(m − 2) . . . 2      2
      m odd:                                               u = 2w, dv = ew dw
           π/2
                                                           du = 2dw, v = ew
                 sinm xdx
       0                                                   I = 2wew − 2                   ew dw
        (m − 1)(m − 3) . . . 2                                                     √ √       √
      =
          m(m − 2) . . . 3                                     = 2wew − 2ew + c = 2 xe x − 2e x + c

 40. m even:                                           45. Let u = sin(ln x), dv = dx
           π/2                                                           dx
                 cosm xdx                                  du = cos(ln x) , v = x
       0
                                                                          x
           π(n − 1)(n − 3)(n − 5) · · · 1                  I = sin(ln x)dx
      =
              2n(n − 2)(n − 4) · · · 2
      m odd:                                                   = x sin(ln x) −              cos(ln x)dx
6.2. INTEGRATION BY PARTS                                                                                                  369

    Integration by parts again,                              = 3 u2 eu − 2                   ueu du
    u = cos(ln x), dv = dx
                     dx
    du = − sin(ln x) , v = x                                 = 3u2 eu − 6 ueu −                      eu du
                      x
       cos(ln x)dx                                           = 3u2 eu − 6ueu + 6eu + c
                                                                       8       √                2
                                                                               3   x
                                                         Hence             e           dx =         3u2 eu du
    = x cos(ln x) +       sin(ln x)dx                              0                           0
                                                                                                    2
    I = x sin(ln x) − x cos(ln x) − I                    = 3u e − 6ueu + 6e
                                                                 2 u                           u
                                                                                                    0
                                                                                                        = 6e2 − 6
    2I = x sin(ln x) − x cos(ln x) + c1
        1               1                            50. Let u = tan−1 x, dv = xdx
    I = x sin(ln x) − x cos(ln x) + c                            dx        x2
        2               2                                du =         , v=
                                                               1 + x2       2
46. Let u = 4 + x2 , du = 2xdx                           I=      x tan−1 xdx
    I=     x ln(4 + x2 )dx                                             x2                1        x2
                                                             = tan−1 x    −                            dx
       1               1                                               2                 2      1 + x2
      =     ln udu = (u ln u − u) + C                                  x2
       2               2                                     = tan−1 x
       1                                                               2
      = [(4 + x ) ln(4 + x2 ) − 4 − x2 ] + c
                2
                                                                1                                1
       2                                                     −       1dx −                           dx
                                                                2                             1 + x2
47. Let u = e2x , du = 2e2x dx                                         x2                1
                             1                               = tan−1 x −                   x − tan−1 x + C
    I = e6x sin(e2x )dx =        u2 sin udu                            2                 2
                             2                                    −1 x
                                                                        2
                                                                                         x 1
    Let v = u2 , dw = sin udu                                = tan x −                     + tan−1 x + c
                                                                       2                 2 2
    dv = 2udu, w = − cos u                                             1
        1                                                Hence             x tan−1 xdx
    I=      −u2 cos u + 2 u cos udu                                0
        2                                                                      x2  x 1
                                                                                                                1
                                                                                                                        π 1
          1                                              =     tan−1 x            − + tan−1 x                       =     −
      = − u2 cos u + u cos udu                                                 2   2 2                          0       4   2
          2
          1 2                                        51. n times. Each integration reduces the power of
      = − u cos u + (u sin u + cos u) + c
          2                                              x by 1.
          1
      = − e4x cos(e2x ) + e2x sin(e2x )
          2                                          52. 1 time. The first integration by parts gets rid
            + cos(e2x ) + c                              of the ln x and turns the integrand into a sim-
                                                         ple integral. See, for example, Problem 4.
               √                     1 −2/3
48. Let u =    3
                   x = x1/3 , du =     x    dx,      53. (a) As the given problem, x sin x2 dx can
                                     3                       be simplified by substituting x2 = u, we
    3u2 du = dx
                                                             can solve the example using substitution
    I = cos x1/3 dx = 3          u2 cos udu                  method.
    Let v = u2 , dw = cos udu                             (b) As the given integral, x2 sin x dx can not
    dv = 2udu, w = sin u                                      be simplified by substitution method and
    I = 3 u2 sin u − 2        u sin udu                       can be solved using method of integration
                                                              by parts.
      = 3u2 sin u − 6       u sin udu                     (c) As the integral, x ln x dx can not be sim-
                                                              plified by substitution and can be solved
      = 3u2 sin u − 6 −u cos u +           cos udu            using the method of integration by parts.
           3                                                                              ln x
      = 3u sin u + 6u√ u −√ sin u + c √
               √       cos      6                         (d) As the given problem,            dx can be
      = 3x sin 3 x + 6 3 x cos 3 x − 6 sin 3 x + c                                         x
                                                              simplified by substituting , ln x = u we
               √                     1 −2/3                   can solve the example by substitution
49. Let u =    3
                   x = x1/3 , du =     x    dx,               method.
                                     3
    3u2 du = dx
            √
            3                                        54. (a) As this integral, x3 e4x dx can not be
    I = e x dx = 3           u2 eu du                        simplified by substitution method and can
370                                                        CHAPTER 6. INTEGRATION TECHNIQUES

           be solved by using the method of integra-      59.
           tion by parts.                                                         e2x
                                        4                                4        2x
       (b) As the given problem, x3 ex dx can be                   x           e /2      +
           simplified by substituting x4 = u, we can               4x3          e2x /4    −
           solve the example using the substitution              12x2          e2x /8    +
                                                                               2x
           method.                                                24x         e /16      −
                                              4                    24         e2x /32    +
        (c) As the given problem,     x−2 e x dx can be
                                     1
           simplified by substituting = u, we can                    x4 e2x dx
                                     x
           solve the example using the substitution                   x4        3x2   3x 3
                                                                =        − x3 +     −    +               e2x + c
           method.                                                    2          2     2   4
       (d) As this integral, x2 e−4x dx can not be        60.
           simplified by substitution and can be                                     cos 2x
           solved by using the method of integration               x     5
                                                                                 sin 2x/2        +
           by parts.
                                                                  5x4          − cos 2x/4        −
 55. First column: each row is the derivative of the             20x3          − sin 2x/8        +
     previous row; Second column: each row is the                60x2           cos 2x/16        −
     antiderivative of the previous row.                         120x           sin 2x/32        +
                                                                  120         − cos 2x/64        −
 56.
                 sin x
           4                                                        x5 cos 2xdx
          x    − cos x      +
         4x3   − sin x      −                                       1 5              5
                                                                =     x sin 2x + x4 cos 2x
        12x2     cos x      +                                       2                4
         24x     sin x      −                                          20              60
                                                                    − x3 sin 2x − x2 cos 2x
          24   − cos x      +                                           8              16
                                                                       120             120
         x4 sin xdx                                                 +      x sin 2x +      cos 2x + c
                                                                        32              64
       = −x4 cos x + 4x3 sin x + 12x2 cos x               61.
         − 24x sin x − 24 cos x + c                                      e−3x
                                                                     3             −3x
 57.                                                              x   −e    /3               +
                 cos x                                           3x2   e−3x /9               −
          x4     sin x      +                                     6x −e−3x /27               +
         4x3   − cos x      −                                      6  e−3x /81               −
        12x2   − sin x      +
                                                                    x3 e−3x dx
         24x     cos x      −
          24     sin x      +                                                x3   x2   2x    2
                                                                =    −          −    −    −           e−3x + c
                                                                             3    3     9   27
         x4 cos xdx
                                                          62.
       = x4 sin x + 4x3 cos x − 12x2 sin x                                 x2
         − 24x cos x + 24 sin x + c                               ln x  x3 /3 +
                                                                    −1
 58.                                                              x    x4 /12 +
               ex                                                −x −2
                                                                       x5 /60 +
           4
          x    ex       +                                       The table will never terminate.
         4x3   ex       −
        12x2   ex       +                                 63. (a) Use the identity
         24x   ex       −                                         cos A cos B
                                                                     1
          24   ex       +                                         = [cos(A − B) + cos(A + B)]
                                                                     2
         x4 ex dx                                                 This identity gives
                                                                              π
           4        3       2
       = (x − 4x + 12x − 24x + 24)e + c  x                                        cos(mx) cos(nx)dx
                                                                             −π
6.2. INTEGRATION BY PARTS                                                                                              371

                π                                                        nπ
                1                                                1
        =         [cos((m − n)x)                             =                 cos2 udu
             −π 2                                                n       −nπ
            + cos((m + n)x)]dx                                                                    nπ
                                                                 1       1    1
            1 sin((m − n)x)                                  =             u + cos(2u)                     =π
        =                                                        n       2    4                   −nπ
            2     m−n                                                            π
                                  π                          And then                sin2 (nx)dx
              sin((m + n)x)                                                     −π
           +                                                         π
                  m+n        −π                              =           (1 − cos2 (nx))dx
        =0                                                       −π
                                                                  π                  π
        It is important that m = n because oth-              =           dx −            cos2 (nx)dx
        erwise cos((m − n)x) = cos 0 = 1                         −π               −π

    (b) Use the identity                                     = 2π − π = π
        sin A sin B
           1                                      65. The only mistake is the misunderstanding of
        = [cos(A − B) − cos(A + B)]
           2                                          antiderivatives. In this problem,                           ex e−x dx
        This identity gives
            π                                         is understood as a group of antiderivatives of
                sin(mx) sin(nx)dx                     ex e−x , not a fixed function. So the subtraction
         −π
                π
                     1                                by      ex e−x dx on both sides of
        =              [cos((m − n)x)
                −π   2
                                                           ex e−x dx = −1 +               ex e−x dx
            − cos((m + n)x)]dx
            1 sin((m − n)x)                           does not make sense.
        =                                                      π √                                    π
            2     m−n
                                  π
                                                  66. V = π     (x sin x)2 dx = π                         x2 sin xdx
                sin((m + n)x)                                    0                                0
            −
                    m+n           −π
                                                      Using integration by parts twice we get
        =0                                                 x2 sin xdx
        It is important that m = n because oth-
        erwise cos((m − n)x) = cos 0 = 1              = −x2 cos x + 2                x cos xdx

64. (a) Use the identity                              = −x2 cos x + 2(x sin x −                  sin xdx)
        cos A sin B                                   = −x2 cos x + 2x sin x + 2 cos x + c
           1
        = [sin(B + A) − sin(B − A)]                   Hence,
           2                                                                                                  π
        This identity gives                           V = (−x2 cos x + 2x sin x + 2 cos x)                    0
            π                                         = π 2 − 4 ≈ 5.87
                cos(mx) sin(nx) dx
         −π
                π
                     1                            67. Let u = ln x, dv = ex dx
        =              [sin((n + m)x)                       dx
                −π   2                                du =     , v = ex
                                                             x
            − sin((n − m)x)] dx                                                               ex
                                                        ex ln xdx = ex ln x −                    dx
            1   cos((n + m)x)                                                                 x
        =     −                                                                 ex
            2       n+m                                    ex ln xdx +             dx = ex ln x + C
                                   π                                            x
                cos((n − m)x)                         Hence,
            +
                    n−m                                                    1
                                   −π                      ex ln x +            dx = ex ln x + c
        =0                                                                 x
    (b) We have seen that                         68. We can guess the formula:
                      1     1
          cos2 xdx = x + cos(2x) + c                       ex (f (x) + f (x))dx = ex f (x) + c
                      2     4
        Hence by letting u = nx:                      and prove it by taking the derivative:
            π
                cos2 (nx)dx                            d x
                                                         (e f (x)) = ex f (x) + ex f (x)
         −π                                           dx
372                                                                                                      CHAPTER 6. INTEGRATION TECHNIQUES

                                                                                                                                                          b
                                            = ex (f (x) + f (x))                                                                             +                f (x) (b − x) dx
                                                                                                                                                      a
                                            1                                                                                            b
 69. Consider,                                  f (x)g (x) dx                                                Consider                        x sin (b − x) dx
                                        0                                                                                            0
      Choose u = g (x) and dv = f (x)dx,                                                                               b                                                        b

      so that du = g (x) dx and , v = f (x) .                                                                =             (b − x) sin xdx =                                        (sin x) (b − x) dx
                                                                                                                   0                                                        0
      Hence, we have                                                                                         Now, consider
           1
               g (x)f (x)dx                                                                                  f (x) = x − sin x ⇒ f (x) = 1 − cos x
       0                                                                                                     and f (x) = sin x.
                                                                1
                                    1
      = g (x) f                 (x)|0               −               f (x)g (x) dx                            Therefore, using
                                                            0                                                f (b) = f (a) + f (a) (b − a)
      = (g (1) f (1) − g (0) f (0))                                                                                                                               b
                                    1                                                                                                            +                    f (x) (b − x) dx,
                       −                g (x)f (x) dx                                                                                                         a
                                0                                                                            we get
      From the given data.                                                                                   b − sin b = 0 − sin 0 + f (0) (b − 0)
                                                1
                                                                                                                                                                  b
      = (0 − 0) −                                   g (x)f (x) dx.                                                                               +                    (sin x) (b − x) dx
                                            0
                                                                                                                                                              0
      Choose, u = g (x) and dv = f (x)dx,                                                                                                                     b
      so that,du = g (x) dx and v = f (x) .                                                                  ⇒ |sin b − b| =                                      x sin (b − x) dx .
                                                                                                                                                      0
      Hence, we have
                   1                                                                                         Further,
      −                g (x)f (x) dx                                                                                                              b                                                b
               0                                                                                             |sin b − b| =                            x sin (b − x) dx ≤                               xdx ,
                                                                            1                                                                 0                                                0
                                                    1
      =−               g (x) f                  (x)|0       −                   f (x) g (x) dx               as sin (b − x) ≤ 1.
                                                                        0
      = − {(g (1) f (1) − g (0) f (0) )                                                                                         b2
                                                                                                             Thus, |sin b − b| ≤ .
                           1                                                                                                    2
           −                   f (x) g (x) dx                                                                Therefore the error in the approximation
                       0                                                                                                           1
      From the given data.                                                                                   sin x ≈ x is at most x2 .
                                                            1                                                                      2
      = − (0 − 0 ) −                                            f (x) g (x) dx
                   1
                                                        0
                                                                                                       6.3        Trigonometric
      =                f (x) g (x) dx.
               0
                                                                                                                  Techniques of
 70. Consider,                                                                                                    Integration
           b                                                                    b
               f (x) (b − x) dx =                                                   (b − x) f (x) dx    1. Let u = sin x, du = cos xdx
       a                                                                    a
      Choose u = (b − x) and dv = f (x) dx,                                                                       cos x sin4 xdx =                                    u4 du
      so that du = −dx and v = f (x) .                                                                            1 5      1
                                                                                                             =      u + c = sin5 x + c
      Hence, we have:                                                                                             5        5
           b
               (b − x)f (x) dx                                                                          2. Let u = sin x, du = cos xdx
       a
                                                                        b                                         cos3 x sin4 xdx =                                    (1 − u2 )u4 du
                                            b
      = (b − x) f                       (x)|a           +                   f (x) dx
                                                                    a                                          u5    u7
                                                                                    b                        =     −    +c
                                                                                                                5    7
      = (0 − [(b − a) f (a)]) +                                                         f (x) dx               sin x sin7 x
                                                                                                                   5
                                                                               a
                                                                                b
                                                                                                             =       −      +c
      = − [(b − a) f (a)] + f                                               (x)|a                                 5      7
      = − [(b − a) f (a)] + f (b) − f (a)                                                               3. Let u = sin 2x, du = 2 cos 2xdx.
           b                                                                                                      π/4
               f (x) (b − x) dx                                                                                            cos 2xsin3 2xdx
       a                                                                                                      0
      = − [(b − a) f (a)] + f (b) − f (a)                                                                                      1                                        1
                                                                                                                  1                                   1 u4                           1
                                                                                                             =                     u3 du =                                  =
      f (b) = f (a) + (b − a) f (a)                                                                               2        0                          2 4               0            8
Ism et chapter_6
Ism et chapter_6
Ism et chapter_6
Ism et chapter_6
Ism et chapter_6
Ism et chapter_6
Ism et chapter_6
Ism et chapter_6
Ism et chapter_6
Ism et chapter_6
Ism et chapter_6
Ism et chapter_6
Ism et chapter_6
Ism et chapter_6
Ism et chapter_6
Ism et chapter_6
Ism et chapter_6
Ism et chapter_6
Ism et chapter_6
Ism et chapter_6
Ism et chapter_6
Ism et chapter_6
Ism et chapter_6
Ism et chapter_6
Ism et chapter_6
Ism et chapter_6
Ism et chapter_6
Ism et chapter_6
Ism et chapter_6
Ism et chapter_6
Ism et chapter_6
Ism et chapter_6
Ism et chapter_6
Ism et chapter_6

Weitere ähnliche Inhalte

Was ist angesagt?

2010 mathematics hsc solutions
2010 mathematics hsc solutions2010 mathematics hsc solutions
2010 mathematics hsc solutionsjharnwell
 
ตัวอย่างข้อสอบเก่า วิชาคณิตศาสตร์ ม.6 ปีการศึกษา 2553
ตัวอย่างข้อสอบเก่า วิชาคณิตศาสตร์ ม.6 ปีการศึกษา 2553ตัวอย่างข้อสอบเก่า วิชาคณิตศาสตร์ ม.6 ปีการศึกษา 2553
ตัวอย่างข้อสอบเก่า วิชาคณิตศาสตร์ ม.6 ปีการศึกษา 2553Destiny Nooppynuchy
 
11X1 T11 06 tangents and normals II
11X1 T11 06 tangents and normals II11X1 T11 06 tangents and normals II
11X1 T11 06 tangents and normals IINigel Simmons
 
11X1 T12 06 tangents & normals II
11X1 T12 06 tangents & normals II11X1 T12 06 tangents & normals II
11X1 T12 06 tangents & normals IINigel Simmons
 
009 solid geometry
009 solid geometry009 solid geometry
009 solid geometryphysics101
 
Lesson 27: Integration by Substitution, part II (Section 10 version)
Lesson 27: Integration by Substitution, part II (Section 10 version)Lesson 27: Integration by Substitution, part II (Section 10 version)
Lesson 27: Integration by Substitution, part II (Section 10 version)Matthew Leingang
 
UPSEE - Mathematics -1998 Unsolved Paper
UPSEE - Mathematics -1998 Unsolved PaperUPSEE - Mathematics -1998 Unsolved Paper
UPSEE - Mathematics -1998 Unsolved PaperVasista Vinuthan
 
Formulario de calculo
Formulario de calculoFormulario de calculo
Formulario de calculoHenry Romero
 
On the Stick and Rope Problem - Draft 1
On the Stick and Rope Problem - Draft 1On the Stick and Rope Problem - Draft 1
On the Stick and Rope Problem - Draft 1Iwan Pranoto
 
Yr.12 Transition Workshop 2012-2013
Yr.12 Transition Workshop 2012-2013Yr.12 Transition Workshop 2012-2013
Yr.12 Transition Workshop 2012-2013rdk.rdk
 

Was ist angesagt? (17)

Add Maths 2
Add Maths 2Add Maths 2
Add Maths 2
 
2010 mathematics hsc solutions
2010 mathematics hsc solutions2010 mathematics hsc solutions
2010 mathematics hsc solutions
 
calculo vectorial
calculo vectorialcalculo vectorial
calculo vectorial
 
ตัวอย่างข้อสอบเก่า วิชาคณิตศาสตร์ ม.6 ปีการศึกษา 2553
ตัวอย่างข้อสอบเก่า วิชาคณิตศาสตร์ ม.6 ปีการศึกษา 2553ตัวอย่างข้อสอบเก่า วิชาคณิตศาสตร์ ม.6 ปีการศึกษา 2553
ตัวอย่างข้อสอบเก่า วิชาคณิตศาสตร์ ม.6 ปีการศึกษา 2553
 
11X1 T11 06 tangents and normals II
11X1 T11 06 tangents and normals II11X1 T11 06 tangents and normals II
11X1 T11 06 tangents and normals II
 
Chapter 08
Chapter 08Chapter 08
Chapter 08
 
AMU - Mathematics - 1999
AMU - Mathematics  - 1999AMU - Mathematics  - 1999
AMU - Mathematics - 1999
 
Stepenovanje
StepenovanjeStepenovanje
Stepenovanje
 
11X1 T12 06 tangents & normals II
11X1 T12 06 tangents & normals II11X1 T12 06 tangents & normals II
11X1 T12 06 tangents & normals II
 
009 solid geometry
009 solid geometry009 solid geometry
009 solid geometry
 
Lesson 27: Integration by Substitution, part II (Section 10 version)
Lesson 27: Integration by Substitution, part II (Section 10 version)Lesson 27: Integration by Substitution, part II (Section 10 version)
Lesson 27: Integration by Substitution, part II (Section 10 version)
 
UPSEE - Mathematics -1998 Unsolved Paper
UPSEE - Mathematics -1998 Unsolved PaperUPSEE - Mathematics -1998 Unsolved Paper
UPSEE - Mathematics -1998 Unsolved Paper
 
Formulario de calculo
Formulario de calculoFormulario de calculo
Formulario de calculo
 
On the Stick and Rope Problem - Draft 1
On the Stick and Rope Problem - Draft 1On the Stick and Rope Problem - Draft 1
On the Stick and Rope Problem - Draft 1
 
Lesson 22: Graphing
Lesson 22: GraphingLesson 22: Graphing
Lesson 22: Graphing
 
Yr.12 Transition Workshop 2012-2013
Yr.12 Transition Workshop 2012-2013Yr.12 Transition Workshop 2012-2013
Yr.12 Transition Workshop 2012-2013
 
Simultaneous eqn2
Simultaneous eqn2Simultaneous eqn2
Simultaneous eqn2
 

Ähnlich wie Ism et chapter_6

Integral table
Integral tableIntegral table
Integral tablebags07
 
Antiderivatives nako sa calculus official
Antiderivatives nako sa calculus officialAntiderivatives nako sa calculus official
Antiderivatives nako sa calculus officialZerick Lucernas
 
Formulas de taylor
Formulas de taylorFormulas de taylor
Formulas de taylorERICK CONDE
 
solucionario de purcell 1
solucionario de purcell 1solucionario de purcell 1
solucionario de purcell 1José Encalada
 
Maths assignment
Maths assignmentMaths assignment
Maths assignmentNtshima
 
Math refresher
Math refresherMath refresher
Math refresherdelilahnan
 
Prim1_secx
Prim1_secxPrim1_secx
Prim1_secxofisico
 
S101-52國立新化高中(代理)
S101-52國立新化高中(代理)S101-52國立新化高中(代理)
S101-52國立新化高中(代理)yustar1026
 
Lesson 8: Derivatives of Logarithmic and Exponential Functions (worksheet sol...
Lesson 8: Derivatives of Logarithmic and Exponential Functions (worksheet sol...Lesson 8: Derivatives of Logarithmic and Exponential Functions (worksheet sol...
Lesson 8: Derivatives of Logarithmic and Exponential Functions (worksheet sol...Matthew Leingang
 

Ähnlich wie Ism et chapter_6 (20)

Lista de derivadas e integrais
Lista de derivadas e integraisLista de derivadas e integrais
Lista de derivadas e integrais
 
Integral table
Integral tableIntegral table
Integral table
 
Sect1 4
Sect1 4Sect1 4
Sect1 4
 
Antiderivatives nako sa calculus official
Antiderivatives nako sa calculus officialAntiderivatives nako sa calculus official
Antiderivatives nako sa calculus official
 
125 5.2
125 5.2125 5.2
125 5.2
 
Formulas de taylor
Formulas de taylorFormulas de taylor
Formulas de taylor
 
Chapter 04
Chapter 04Chapter 04
Chapter 04
 
Chapter 07
Chapter 07Chapter 07
Chapter 07
 
solucionario de purcell 1
solucionario de purcell 1solucionario de purcell 1
solucionario de purcell 1
 
Maths assignment
Maths assignmentMaths assignment
Maths assignment
 
Math refresher
Math refresherMath refresher
Math refresher
 
Prim1_secx
Prim1_secxPrim1_secx
Prim1_secx
 
Double integration
Double integrationDouble integration
Double integration
 
S101-52國立新化高中(代理)
S101-52國立新化高中(代理)S101-52國立新化高中(代理)
S101-52國立新化高中(代理)
 
01 regras diferenciacao
01   regras diferenciacao01   regras diferenciacao
01 regras diferenciacao
 
Regras diferenciacao
Regras diferenciacaoRegras diferenciacao
Regras diferenciacao
 
Sect1 2
Sect1 2Sect1 2
Sect1 2
 
Lesson 22: Graphing
Lesson 22: GraphingLesson 22: Graphing
Lesson 22: Graphing
 
answers tutor 8
answers tutor 8answers tutor 8
answers tutor 8
 
Lesson 8: Derivatives of Logarithmic and Exponential Functions (worksheet sol...
Lesson 8: Derivatives of Logarithmic and Exponential Functions (worksheet sol...Lesson 8: Derivatives of Logarithmic and Exponential Functions (worksheet sol...
Lesson 8: Derivatives of Logarithmic and Exponential Functions (worksheet sol...
 

Mehr von Drradz Maths

Mehr von Drradz Maths (20)

Figures
FiguresFigures
Figures
 
Formulas
FormulasFormulas
Formulas
 
Revision 7.1 7.3
Revision 7.1 7.3Revision 7.1 7.3
Revision 7.1 7.3
 
Tutorial 9
Tutorial 9Tutorial 9
Tutorial 9
 
Tutorial 8
Tutorial 8Tutorial 8
Tutorial 8
 
MTH3101 Tutor 7 lagrange multiplier
MTH3101  Tutor 7 lagrange multiplierMTH3101  Tutor 7 lagrange multiplier
MTH3101 Tutor 7 lagrange multiplier
 
Tutorial 6 en.mughti important
Tutorial 6 en.mughti importantTutorial 6 en.mughti important
Tutorial 6 en.mughti important
 
Figures
FiguresFigures
Figures
 
Formulas
FormulasFormulas
Formulas
 
Figures
FiguresFigures
Figures
 
First_attachment MTH3101
First_attachment MTH3101First_attachment MTH3101
First_attachment MTH3101
 
Tutorial 9 mth 3201
Tutorial 9 mth 3201Tutorial 9 mth 3201
Tutorial 9 mth 3201
 
Tutorial 8 mth 3201
Tutorial 8 mth 3201Tutorial 8 mth 3201
Tutorial 8 mth 3201
 
Tutorial 7 mth 3201
Tutorial 7 mth 3201Tutorial 7 mth 3201
Tutorial 7 mth 3201
 
Tutorial 6 mth 3201
Tutorial 6 mth 3201Tutorial 6 mth 3201
Tutorial 6 mth 3201
 
Tutorial 5 mth 3201
Tutorial 5 mth 3201Tutorial 5 mth 3201
Tutorial 5 mth 3201
 
Tutorial 4 mth 3201
Tutorial 4 mth 3201Tutorial 4 mth 3201
Tutorial 4 mth 3201
 
Tutorial 2, 2(e), no. 7
Tutorial 2,  2(e),  no. 7Tutorial 2,  2(e),  no. 7
Tutorial 2, 2(e), no. 7
 
Tutorial 3 mth 3201
Tutorial 3 mth 3201Tutorial 3 mth 3201
Tutorial 3 mth 3201
 
Tutorial 2 mth 3201
Tutorial 2 mth 3201Tutorial 2 mth 3201
Tutorial 2 mth 3201
 

Ism et chapter_6

  • 1. 1 6. sec 2t tan 2t dt = sec 2t + c 2 7. (x2 + 4)2 dx = (x4 + 8x2 + 16)dx x5 8 = + x3 + 16x + c 5 3 Chapter 6 8. x(x2 + 4)2 dx = (x5 + 8x3 + 16x)dx x6 = + 2x4 + 8x2 + c Integration 3 6 3 x 9. dx = tan−1 + c Techniques 16 + x2 2 4 1 4 10. 2 dx = tan−1 x + c 4 + 4x 2 1 11. √ dx 3 − 2x − x2 6.1 Review of Formulas 1 x+1 = dx = arcsin +c and Techniques 4 − (x + 1)2 2 1 ax x+1 1. eax dx = e + c, for a = 0. 12. √ dx a 3 − 2x − x2 1 −2(x + 1) =− dx 1 2 4 − (x + 1)2 2. cos(ax)dx = sin(ax) + c, for a = 0. a 1 = − · 2[4 − (x + 1)2 ]1/2 + C 2 1 1 1 3. √ dx = dx = − 4 − (x + 1)2 + c a2 − x2 x 2 a 1− a 4 x 1 13. dx Let u = , du = dx. 5 + 2x + x2 a a 1 x+1 1 =4 dx = 2 tan−1 +c = √ du = sin−1 (u) + c 4 + (x + 1)2 2 1 − u2 −1 x = sin + c, a > 0. 4x + 4 a 14. dx 5 + 2x + x2 b 2(x + 1) 4. √ dx =2 dx = 2 ln | 4 + (x + 1)2 | + c |x| x2 − a2 4 + (x + 1)2 b 1 4t = dx 15. dt x 2 a 5 + 2t + t2 |x| a −1 4t + 4 4 x 1 = dt − dt Let u = , du = dx and |au| = |x| . 5 + 2t + t2 5 + 2t + t2 a a t+1 b = 2 ln 4 + (t + 1) 2 − 2tan−1 +c = √ du 2 |au| u2 − 1 b 1 t+1 2 (t + 1) = √ du 16. dt = dt |a| |u| u2 − 1 t2 + 2t + 4 2 (t + 1) + 3 b 1 = sec−1 (u) + c 2 = ln (t + 1) + 3 + c |a| 2 b x = sec−1 + c, a > 0. 17. 1 e3−2x dx = − e3−2x + c |a| a 2 1 3 5. sin(6t)dt = − cos(6t) + c 18. 3e−6x dx = − e−6x + c 6 6 360
  • 2. 6.1. REVIEW OF FORMULAS AND TECHNIQUES 361 2 −1/3 30. Let u = ex , du = ex dx 19. Let u = 1 + x2/3 , du = x dx 3 ex 1 4 3 √ dx = √ du dx = 4 u−1 du 1 − e2x 1 − u2 x1/3 (1 + x2/3 ) 2 = sin−1 u + C = sin−1 ex + c 2/3 = 6 ln |u| + C = 6 ln |1 + x |+c 31. Let u = x2 , du = 2xdx 3 x 1 1 20. Let u = 1 + x3/4 , du = x−1/4 dx √ dx = √ du 4 1 − x4 2 1 − u2 2 2 dx = dx 1 1 x1/4 + x x1/4 (1 + x3/4 ) = sin−1 u + C = sin−1 x2 + c 4 8 2 2 =2 u−1 du = ln |u| + C 3 3 32. Let u = 1 − x4 , du = −4x3 dx 8 2x3 1 = ln |1 + x3/4 | + c √ dx = − u−1/2 du 3 1−x 4 2 √ 1 = −u1/2 + C = −(1 − x4 )1/2 + c 21. Let u = x, du = √ dx √ 2 x sin x 1+x √ dx = 2 sin udu 33. dx x 1 + x2 √ 1 1 2x = −2 cos u + C = −2 cos x + c = dx + dx 1 + x2 2 1 + x2 1 1 1 22. Let u = , du = − 2 dx = tan−1 x + ln |1 + x2 | + c x x 2 cos(1/x) dx = − cos udu 1 x2 34. √ dx 1 x+x = − sin u + C = − sin + c x 1 = x−1/2 · dx 23. Let u = sin x, du = cos xdx 1 + x1/2 π 0 = 2 ln | 1 + x1/2 | + c cos xesin x dx = eu du = 0 0 0 ln x2 1 2 35. dx = 2 ln x dx 24. Let u = tan x, du = sec xdx x x π/2 1 1 sec2 xetan x dx = eu du Let u = ln x, du = dx. x 0 0 2 u 1 =2 u du = u2 + c = (ln x) + c =e =e−1 0 3 3 3 0 x3 26 25. sec x tan xdx 36. e2 ln x dx = x2 dx = = −π/4 1 1 3 1 3 0 √ 4 = sec x =1− 2 √ −π/4 37. x x − 3dx 3 π/2 4 √ π/2 26. csc2 xdx = − cot x =1 = (x − 3 + 3) x − 3dx π/4 π/4 3 4 4 3 2 = (x − 3)3/2 dx + 3 (x − 3)1/2 dx 27. Let u = x , du = 3x dx 3 3 x2 1 1 2 4 2 4 12 dx = du = (x − 3)5/2 + 3 · (x − 3)3/2 = 1 + x6 3 1 + u2 5 3 5 3 3 1 1 = tan−1 u + C = tan−1 x3 + c 1 3 3 38. x(x − 3)2 dx 0 x5 1 1 28. dx = ln(1 + x6 ) + c = (x3 − 6x2 + 9x)dx 1 + x6 6 0 1 1 x x4 9 11 29. √ dx = sin−1 + c = − 2x3 + x2 = 4−x 2 2 4 2 0 4
  • 3. 362 CHAPTER 6. INTEGRATION TECHNIQUES 4 2 x2 + 1 39. √ dx 47. f (x)dx 1 x 0 4 4 1 2 x x2 = x3/2 dx + x−1/2 dx = 2+1 dx + dx 1 1 0 x 1 x2 + 1 1 2 4 1 1 2 5/2 4 72 = ln |x2 + 1| + 1− 2 dx = x + 2x1/2 = 2 0 1 x +1 5 1 1 5 1 2 = ln 2 + (x − arctan x) 0 0 2 1 2 1 −x2 e−4 − 1 ln 2 π 40. xe−x dx = − e = = + 1 + − arctan 2 −2 2 −2 2 2 4 4x + 1 5 5 x 48. dx 41. dx = √ arctan √ + c 2x2 + 4x + 10 3 + x2 3 3 5 4x + 4 3 dx: N/A = 2 + 4x + 10 dx − 2 + 4x + 10 dx 3 + x3 2x 2x 3 1 = ln |2x2 + 4x + 10| − dx 1 2 (x + 1)2 + 4 42. sin(3x)dx = sin(3x)3dx 3 3 x+1 Let u = 3x, du = 3dx. = ln |2x2 + 4x + 10| − tan−1 +c 1 1 4 2 = (sin u)du = − cos u + c 3 3 1 1 49. dx = tan−1 (x) + c. = − cos(3x) + c. (1 + x2 ) 3 x 1 2x 2) dx = dx (1 + x 2 (1 + x2 ) sin3 xdx = (sin2 x) sin xdx 1 = ln 1 + x2 + c. 2 = (1 − cos2 x)sin xdx x2 x2 + 1 − 1 dx = dx Let u = cos x, du = − sin xdx. (1 + x2 ) (1 + x2 ) 2 = 1 − u2 (−du) = u2 du − du x +1 1 = 2 + 1) dx − dx (x (1 + x2 ) u3 cos3 x 1 = −u= − cos x. = dx − dx 3 3 (1 + x2 ) = x − tan−1 (x) + c. 43. ln xdx: N/A x3 1 x2 2) dx = 2xdx Substituting u = ln x, (1 + x 2 (1 + x2 ) ln x 1 dx = ln2 x + c Let u = x2 , du = 2xdx. 2x 4 1 u 1 u+1−1 = du = du 2 1+u 2 1+u 44. Substituting u = x4 1 u+1 1 = du − du x3 1 2 1+u 1+u dx = arctan x4 + c 1 + x8 4 1 1 = du − du x4 2 1+u dx: N/A 1 1 + x8 = (u − ln (1 + u)) + c 2 1 1 45. e−x dx: N/A 2 = x2 − ln 1 + x2 + c. 2 2 Substituting u = −x2 Hence we can generalize this as follows, 2 1 2 xn xe−x dx = − e−x + c 1 + x2 dx 2 1 xn−2 = xn−1 − dx 46. sec xdx: N/A n−1 1 + x2 x 1 1 sec2 xdx = tan x + c 50. dx = 2xdx 1 + x4 2 1 + x4
  • 4. 6.2. INTEGRATION BY PARTS 363 Let u = x2 , du = 2xdx. 1 2x 1 2x xe2x dx =xe − e dx 1 1 1 2 2 = du = tan−1 (u) + c 1 1 2 1 + u2 2 = xe2x − e2x + c. 1 2 4 = tan−1 x2 + c. 2 4. Let u = ln x, dv = x dx x3 1 1 1 x2 4 dx = 4x3 dx du = dx and v = . 1+x 4 1 + x4 x 2 Let u = 1 + x4 , du = 4x3 . 1 1 x ln x dx = x2 ln x − x dx 1 1 1 2 2 = du = ln (u) + c 1 1 4 u 4 = x2 ln x − x2 + c. 1 2 4 = ln 1 + x4 + c. 4 5. Let u = ln x, dv = x2 dx x5 1 1 dx du = dx, v = x3 . 1 + x4 x 3 1 x4 2 1 3 1 3 1 = 2xdx x ln xdx = x ln x − x · dx 2 1 + x4 3 3 x Let u = x2 , du = 2xdx. 1 3 1 = x ln x − x2 dx 1 u2 1 u2 + 1 − 1 3 3 = du = du 1 1 2 1+u 2 2 1 + u2 = x3 ln x − x3 + c. 1 u2 + 1 1 3 9 = du − du 2 1 + u2 1 + u2 1 6. Let u = ln x, du = dx. 1 1 x = du − du ln x u2 1 2 1 + u2 dx = udu = + c = (ln x)2 + c. 1 x 2 2 = u − tan−1 (u) + c 2 1 2 7. Let u = x2 , dv = e−3x dx = x − tan−1 x2 + c. 1 2 du = 2xdx, v = − e−3x Hence we can generalize this as follows, 3 x4n+1 1 x2n−2 x4(n−1)+1 I = x2 e−3x dx dx = − dx 1 + x4 2 n−1 1 + x4 1 1 and = − x2 e−3x − − e−3x · 2xdx 3 3 x4n+3 1 x2n x4(n−1)+3 1 2 dx = − dx = − x2 e−3x + xe−3x dx 1+x 4 4 n 1 + x4 3 3 Let u = x, dv = e−3x dx 1 6.2 Integration by Parts du = dx, v = − e−3x 3 1 I = − x2 e−3x 1. Let u = x, dv = cos xdx 3 du = dx, v = sin x. 2 1 1 + − xe−3x − − e−3x dx x cos xdx = x sin x − sin xdx 3 3 3 1 2 2 = x sin x + cos x + c = − x2 e−3x − xe−3x + e−3x dx 3 9 9 1 2 2 −3x 2. Let u = x, dv = sin 4xdx = − x2 e−3x − xe−3x − e +c 1 3 9 27 du = dx, v = − cos 4x 4 8. Let u = x3 , du = 3x2 dx. 3 1 1 x sin 4x dx x2 ex dx = eu dx = eu + c 3 3 1 1 1 x3 = − x cos 4x − − cos 4x dx = e + c. 4 4 3 1 1 = − x cos 4x + sin 4x + c. 4 16 9. Let I = ex sin 4xdx 3. Let u = x, dv = e2x dx u = ex , dv = sin 4xdx 1 1 du = dx, v = e2x . du = ex dx, v = − cos 4x 2 4
  • 5. 364 CHAPTER 6. INTEGRATION TECHNIQUES 1 1 1 I = − ex cos 4x − − cos 4x ex dx du = cos xdx v = − cos 2x 4 4 2 1 x 1 1 1 1 = − e cos 4x + x e cos 4xdx I = cos x sin 2x + − cos 2x sin x 4 4 2 2 2 1 Use integration by parts again, this time let − − cos 2x cos xdx u = ex , dv = cos 4xdx 2 1 1 1 1 du = ex dx, v = sin 4x = cos x sin 2x − cos 2x sin x + Idx 4 2 4 4 1 x I = − e cos 4x So, 4 3 1 1 1 1 x 1 I = cos x sin 2x − cos 2x sin x + c1 + e sin 4x − (sin 4x)ex dx 4 2 4 4 4 4 1 1 1 2 1 I = − ex cos 4x + ex sin 4x − I I = cos x sin 2x − cos 2x sin x + c 4 16 16 3 3 So, 17 1 1 12. Here we use the trigonometric identity: I = − ex cos 4x + ex sin 4x + c1 sin 2x = 2 sin x cos x. 16 4 16 4 1 We then make the substitution I = − ex cos 4x + ex sin 4x + c 17 17 u = sin x, du = cos x dx. 10. Let, u = e2x , dv = cos x dx so that, sin x sin 2x dx = 2 sin2 x cos x dx du = 2e2x dx and v = sin x. e2x cos x dx 2 3 2 = 2u2 du = u + c = sin3 x + c 3 3 = e2x sin x − 2 e2x sin x dx This integral can also be done by parts, twice. If this is done, an equivalent answer is ob- Let, u = e2x , dv = sin x dx so that, tained: du = 2e2x dx and v = − cos x. 1 2 cos x sin 2x − cos 2x sin x + c e2x sin x dx 3 3 13. Let u = x, dv = sec2 xdx 2x 2x = −e cos x + 2 e cos x dx du = dx, v = tan x e2x cos x dx x sec2 xdx = x tan x − tan xdx sin x = e2x sin x + 2e2x cos x − 4 e2x cos x dx = x tan x − dx cos x Now we notice that the integral on both of Let u = cos x, du = − sin xdx 1 these is the same, so we bring them to one side x sec2 xdx = x tan x + du of the equation. u = x tan x + ln |u| + c 5 e2x cos x dx = x tan x + ln |cos x| + c = e2x sin x + 2e2x cos x + c1 14. Let u = (ln x)2 , dv = dx ln x e2x cos x dx du = 2 dx, v = x x 1 2x 2 = e sin x + e2x cos x + c I = (ln x)2 dx 5 5 ln x = x(ln x)2 − x·2 dx 11. Let I = cos x cos 2xdx x and u = cos x, dv = cos 2xdx = x(ln x)2 − 2 ln xdx 1 du = sin xdx, v = sin 2x Integration by parts again, 2 1 1 1 u = ln x, dv = dxdu = dx, v = x I = cos x sin 2x − sin 2x(− sin x)dx x 2 2 1 2 1 1 I = x(ln x) − 2 x ln x − x · dx = cos x sin 2x + sin x sin 2xdx x 2 2 Let,u = sin x, dv = sin 2xdx = x(ln x)2 − 2x ln x + 2 dx
  • 6. 6.2. INTEGRATION BY PARTS 365 = x(ln x)2 − 2x ln x + 2x + c 20. Let u = 2x, dv = cos x dx duπ= 2 dxandv = sin x. π 2 15. Let u = x2 , dv = xex dx so that, du = 2x dx 2x cos xdx = 2x sin x|0 − 2 π sin xdx 1 2 0 0 and v = ex (v is obtained using substitu- = (2x sin x + π 2 cos x)|0 = −4. 2 tion). 2 1 2 2 1 x3 ex dx = x2 ex − xex dx 21. x2 cos πxdx 2 0 1 2 1 2 = x2 ex − ex + c Let u = x2 , dv = cos πxdx, 2 2 sin πx du = 2xdx, v = . π x 1 1 1 16. Let u = x2 , dv = dx sin πx sin πx 3/2 x2 cos πxdx = x2 − 2xdx (4 + x2 ) 0 π 0 0 π 1 2 1 du = 2xdx, v = − √ = (0 − 0) − x sin (πx) dx 4 + x2 π 0 3 1 x x 2 3/2 dx = x2 3/2 dx =− x sin (πx) dx (4 + x2 ) (4 + x2 ) π 0 x2 1 Let u = x, dv = sin(πx)dx, = −√ + √ 2xdx cos(πx) 4+x 2 4 + x2 du = dx, v = − . x2 π 1 =− + 2 (4 + x2 ) + c. 2 (4 + x2 ) − xsin(πx)dx π 0 1 1 17. Let u = ln(sin x), dv = cos xdx 2 x cos(πx) cos(πx) =− − − − dx 1 π π 0 0 π du = · cos xdx, v = sin x 1 sin x 2 cos π 1 sin(πx) =− (− − 0) + I = cos x ln(sin x)dx π π π π 0 = sin x ln(sin x) 2 1 1 2 1 =− + (0 − 0) = − 2 − sin x · · cos xdx π π π π sin x 1 = sin x ln(sin x) − cos xdx 22. x2 e3x dx = sin x ln(sin x) − sin x + c 0 Let u = x2 , dv = e3x dx, e3x 18. This is a substitution u = x2 . du = 2xdx, v = . 1 3 x sin x2 dx = sin udu 1 x2 e3x 1 1 3x e 2 x2 e3x dx = − 2xdx 1 1 3 0 3 = − cos u + c = − cos x2 + c. 0 0 2 2 1 3 2 1 3x = e −0 − xe dx. 19. Let u = x, dv = sin 2xdx 3 3 0 1 Let u = x, dv = e3x dx, du = dx, v = − cos 2x e3x 2 dv = dx, v = . 1 3 x sin 2xdx e3 2 1 3x 0 − xe dx 1 1 1 1 3 3 0 = − x cos 2x − − cos 2x dx e3 2 e3x 1 1 3x e 2 0 0 2 = − x − dx 1 1 1 3 3 3 0 0 3 = − (1 cos 2 − 0 cos 0) + cos 2xdx 1 2 2 0 e3 2 e3 e3x 1 = − − dx 1 1 1 3 3 3 0 3 = − cos 2 + sin 2x 3 1 2 2 2 0 e 2 e3 e3x 1 1 = − − = − cos 2 + (sin 2 − sin 0) 3 3 3 9 0 2 4 e 3 2 e3 1 3 1 1 = − − e −1 = − cos 2 + sin 2 3 3 3 9 2 4
  • 7. 366 CHAPTER 6. INTEGRATION TECHNIQUES e3 2e3 2 3 x cos (ax) sin (ax) = − + e −1 =− + + c, a = 0. 3 9 27 a a2 e3 2e3 2e3 2 5e3 2 = − + − = − 3 9 27 27 27 27 27. (xn ) (ln x) dx = (ln x) (xn ) dx 10 Let u = ln x, dv = xn dx, 23. ln 2xdx 1 xn+1 1 du = dx, v = . Let u = ln 2x, dv = dx x (n + 1) 1 du = dx, v = x. (ln x)(xn ) dx x 10 10 10 1 xn+1 xn+1 dx ln (2x)dx = x ln (2x)|1 − x dx = (ln x) − 1 1 x (n + 1) (n + 1) x 10 = (10 ln(20) − ln 2) − dx xn+1 (ln x) xn = − dx 1 10 (n + 1) (n + 1) = (10 ln(20) − ln 2) − [x]1 x n+1 (ln x) x n+1 = (10 ln(20) − ln 2) − (10 − 1) = − 2 + c, n = −1. (n + 1) (n + 1) = (10 ln(20) − ln 2) − 9. 24. Let, u = ln x, dv = x dx 28. (sin ax) (cos bx) dx 1 x2 du = dx, v = . x 2 Let u = sin ax, dv = (cos bx) dx 2 2 2 sin bx 1 1 du = a (cos ax) dx, v = . x ln xdx = x2 ln x − xdx b 1 2 1 1 2 2 1 2 1 3 sin ax cos bx dx = x ln x − x2 = 2 ln 2 − . 2 4 1 4 sin bx sin bx = (sin ax) − a (cos ax) dx b b 25. x2 eax dx (sin ax) (sin bx) a = − (cos ax) (sin bx) dx b b Let u = x2 , dv = eax dx, Let u = cos ax, dv = sin bxdx, eax cos bx du = 2xdx, v = . du = −a (sin ax) dx, v = − . aax b e eax x2 eax dx = x2 − 2xdx sin ax sin bx a a a − cos ax sin bx dx b b x2 eax 2 sin ax sin bx a − cos bx = − xeax dx. = − cos ax a a b b b Let u = x, dv = eax dx, eax − cos bx dv = dx, v = . − (− sin ax) adx a b 2 ax x e 2 sin ax sin bx a − cos ax cos bx − xeax dx = − a a b b b x2 eax 2 eax eax a = − x − dx − cos bx sin ax dx a a a a b x2 eax 2 xeax eax sin ax sin bx a cos ax cos bx = − − 2 +c = + a a a a b b2 a 2 x2 eax 2xeax 2eax + sin ax cos bx dx = − + 3 + c, a = 0. b a a2 a sin ax cos bx dx 26. x sin (ax) dx sin ax sin bx a cos ax cos bx = + Let u = x, dv = sin axdx, b b2 cos ax a 2 du = dx, v = − . + sin ax cos bx dx a b a 2 xsin (ax) dx sin ax cos bx dx − sin ax cos bx dx b − cos (ax) cos (ax) sin ax sin bx a cos ax cos bx =x − − dx = + a a b b2
  • 8. 6.2. INTEGRATION BY PARTS 367 a2 1− sin ax cos bx dx + (n − 1) sinn xdx b2 sin ax sin bx a cos ax cos bx n sinn xdx = + b b2 = − sinn−1 x cos x sin ax cos bx dx − (n − 1) sinn−2 xdx b2 sin ax sin bx a cos ax cos bx = + 1 b2 − a2 b b2 sinn xdx = − sinn−1 x cos x n sin ax cos bx dx n−1 − sinn−2 xdx 1 n = 2 − a2 (b sin ax sin bx + a cos ax cos bx) , b a = 0 b = 0. 31. x3 ex dx = ex (x3 − 3x2 + 6x − 6) + c 29. Letu = cosn−1 x, dv = cos xdx du = (n − 1)(cosn−2 x)(− sin x)dx, v = sin x 32. cos5 xdx cosn xdx 1 4 = cos4 sin x + cos3 xdx = sin x cos n−1 x 5 5 1 − (sin x)(n − 1)(cosn−2 x)(− sin x)dx = cos4 sin x 5 = sin x cosn−1 x 4 1 2 + cos2 x sin x + cos xdx 5 3 3 + (n − 1)(cosn−2 x)(sin2 x)dx 1 4 = cos4 sin x + cos2 x sin x = sin x cosn−1 x 5 15 8 + (n − 1)(cosn−2 x)(1 − cos2 x)dx + sin x + c 15 = sin x cosn−1 x + (n − 1)(cosn−2 x − cosn x)dx 33. cos3 xdx 1 2 Thus, cosn xdx = cos2 x sin x + cos xdx 3 3 1 2 = sin x cosn−1 x + (n − 1) cosn−2 xdx = cos2 x sin x + sin x + c 3 3 − (n − 1) cosn xdx. 34. sin4 xdx n cosn xdx = sin x cosn−1 x 1 = − sin3 x cos x + 3 sin2 xdx 4 4 + (n − 1) cosn−2 xdx 1 3 1 1 = − sin3 x cos x + x − sin 2x 4 4 2 4 cosn xdx 1 1 n−1 35. x4 ex dx = sin x cosn−1 x + cos n−2 xdx 0 n n 1 = ex (x4 − 4x3 + 12x2 − 24x + 24) 0 30. Let u = sinn−1 x, dv = sin x dx = 9e − 24 du = (n − 1) sinn−2 x cos x, v = − cos x. sinn xdx 36. Using the work done in Exercise 34, π/2 = − sinn−1 x cos x sin4 xdx 0 + (n − 1) cos2 x sinn−2 xdx 1 3 3 π/2 = − sin3 x cos x + x − sin 2x = − sinn−1 x cos x 4 8 16 0 3π + (n − 1) (1 − sin2 x) sinn−2 xdx = 16 = − sinn−1 x cos x π/2 − (n − 1) sinn−2 xdx 37. sin5 xdx 0
  • 9. 368 CHAPTER 6. INTEGRATION TECHNIQUES π/2 π/2 1 4 π/2 3 = − sin4 x cos x + sin xdx cosm xdx 5 0 5 0 0 1 π/2 (n − 1)(n − 3)(n − 5) · · · 2 = − sin4 x cos x = . 5 n(n − 2)(n − 4) · · · 3 0 π/2 4 1 2 41. Let u = cos−1 x, dv = dx + − sin2 x cos x − cos x 1 5 3 3 0 du = − √ dx, v = x (Using Exercise 30) 1 − x2 1 π π cos−1 xdx =− sin4 cos − sin4 0 cos 0 I= 5 2 2 4 1 2 π π 2 π 1 + − sin cos − cos = x cos−1 x − x −√ dx 5 3 2 2 3 2 1 − x2 8 x = = x cos−1 x + dx √ 15 1 − x2 Substituting u = 1 − x2 , du = −2xdx 38. Here we will again use the work we did in Ex- 1 1 ercise 34. I = x cos−1 x + √ − du u 2 sin6 xdx 1 = x cos1 x − u−1/2 du 1 5 2 = − sin5 x cos x + sin4 xdx 1 6 6 = x cos−1 x − · 2u1/2 + c 1 2 = − sin5 x cos x = x cos−1 x − 1 − x2 + c 6 5 1 3 3 + − sin3 x cos x + x − sin 2x + c 42. Let u = tan−1 x, dv = dx 6 4 8 16 1 1 5 du = dx, v = x = − sin5 x cos x − sin3 x cos x 1 + x2 6 24 x 15 + x− 15 sin 2x + c I= tan−1 xdx = x tan−1 x − dx 48 96 1 + x2 Substituting u = 1 + x2 , We now just have to plug in the endpoints: 1 π/2 I = x tan−1 x − ln(1 + x2 ) + c. sin6 xdx 2 0 √ 1 1 5 43. Substituting u = x, du = √ dx = − sin5 x cos x − sin3 x cos x 2 x 6 24 √ π/2 I = sin xdx = 2 u sin udu 15 15 + x− sin 2x 48 96 0 = 2(−u cos u + sin u) + c √ √ √ 15π = 2(− x cos x + sin x) + c = 96 √ 44. Substituting w = x 39. m even : 1 1 π/2 dw = √ dx = dx sinm xdx 2 x 2w √ x 0 (m − 1)(m − 3) . . . 1 π I= e dx = 2wew dx = · Next, using integration by parts m(m − 2) . . . 2 2 m odd: u = 2w, dv = ew dw π/2 du = 2dw, v = ew sinm xdx 0 I = 2wew − 2 ew dw (m − 1)(m − 3) . . . 2 √ √ √ = m(m − 2) . . . 3 = 2wew − 2ew + c = 2 xe x − 2e x + c 40. m even: 45. Let u = sin(ln x), dv = dx π/2 dx cosm xdx du = cos(ln x) , v = x 0 x π(n − 1)(n − 3)(n − 5) · · · 1 I = sin(ln x)dx = 2n(n − 2)(n − 4) · · · 2 m odd: = x sin(ln x) − cos(ln x)dx
  • 10. 6.2. INTEGRATION BY PARTS 369 Integration by parts again, = 3 u2 eu − 2 ueu du u = cos(ln x), dv = dx dx du = − sin(ln x) , v = x = 3u2 eu − 6 ueu − eu du x cos(ln x)dx = 3u2 eu − 6ueu + 6eu + c 8 √ 2 3 x Hence e dx = 3u2 eu du = x cos(ln x) + sin(ln x)dx 0 0 2 I = x sin(ln x) − x cos(ln x) − I = 3u e − 6ueu + 6e 2 u u 0 = 6e2 − 6 2I = x sin(ln x) − x cos(ln x) + c1 1 1 50. Let u = tan−1 x, dv = xdx I = x sin(ln x) − x cos(ln x) + c dx x2 2 2 du = , v= 1 + x2 2 46. Let u = 4 + x2 , du = 2xdx I= x tan−1 xdx I= x ln(4 + x2 )dx x2 1 x2 = tan−1 x − dx 1 1 2 2 1 + x2 = ln udu = (u ln u − u) + C x2 2 2 = tan−1 x 1 2 = [(4 + x ) ln(4 + x2 ) − 4 − x2 ] + c 2 1 1 2 − 1dx − dx 2 1 + x2 47. Let u = e2x , du = 2e2x dx x2 1 1 = tan−1 x − x − tan−1 x + C I = e6x sin(e2x )dx = u2 sin udu 2 2 2 −1 x 2 x 1 Let v = u2 , dw = sin udu = tan x − + tan−1 x + c 2 2 2 dv = 2udu, w = − cos u 1 1 Hence x tan−1 xdx I= −u2 cos u + 2 u cos udu 0 2 x2 x 1 1 π 1 1 = tan−1 x − + tan−1 x = − = − u2 cos u + u cos udu 2 2 2 0 4 2 2 1 2 51. n times. Each integration reduces the power of = − u cos u + (u sin u + cos u) + c 2 x by 1. 1 = − e4x cos(e2x ) + e2x sin(e2x ) 2 52. 1 time. The first integration by parts gets rid + cos(e2x ) + c of the ln x and turns the integrand into a sim- ple integral. See, for example, Problem 4. √ 1 −2/3 48. Let u = 3 x = x1/3 , du = x dx, 53. (a) As the given problem, x sin x2 dx can 3 be simplified by substituting x2 = u, we 3u2 du = dx can solve the example using substitution I = cos x1/3 dx = 3 u2 cos udu method. Let v = u2 , dw = cos udu (b) As the given integral, x2 sin x dx can not dv = 2udu, w = sin u be simplified by substitution method and I = 3 u2 sin u − 2 u sin udu can be solved using method of integration by parts. = 3u2 sin u − 6 u sin udu (c) As the integral, x ln x dx can not be sim- plified by substitution and can be solved = 3u2 sin u − 6 −u cos u + cos udu using the method of integration by parts. 3 ln x = 3u sin u + 6u√ u −√ sin u + c √ √ cos 6 (d) As the given problem, dx can be = 3x sin 3 x + 6 3 x cos 3 x − 6 sin 3 x + c x simplified by substituting , ln x = u we √ 1 −2/3 can solve the example by substitution 49. Let u = 3 x = x1/3 , du = x dx, method. 3 3u2 du = dx √ 3 54. (a) As this integral, x3 e4x dx can not be I = e x dx = 3 u2 eu du simplified by substitution method and can
  • 11. 370 CHAPTER 6. INTEGRATION TECHNIQUES be solved by using the method of integra- 59. tion by parts. e2x 4 4 2x (b) As the given problem, x3 ex dx can be x e /2 + simplified by substituting x4 = u, we can 4x3 e2x /4 − solve the example using the substitution 12x2 e2x /8 + 2x method. 24x e /16 − 4 24 e2x /32 + (c) As the given problem, x−2 e x dx can be 1 simplified by substituting = u, we can x4 e2x dx x solve the example using the substitution x4 3x2 3x 3 = − x3 + − + e2x + c method. 2 2 2 4 (d) As this integral, x2 e−4x dx can not be 60. simplified by substitution and can be cos 2x solved by using the method of integration x 5 sin 2x/2 + by parts. 5x4 − cos 2x/4 − 55. First column: each row is the derivative of the 20x3 − sin 2x/8 + previous row; Second column: each row is the 60x2 cos 2x/16 − antiderivative of the previous row. 120x sin 2x/32 + 120 − cos 2x/64 − 56. sin x 4 x5 cos 2xdx x − cos x + 4x3 − sin x − 1 5 5 = x sin 2x + x4 cos 2x 12x2 cos x + 2 4 24x sin x − 20 60 − x3 sin 2x − x2 cos 2x 24 − cos x + 8 16 120 120 x4 sin xdx + x sin 2x + cos 2x + c 32 64 = −x4 cos x + 4x3 sin x + 12x2 cos x 61. − 24x sin x − 24 cos x + c e−3x 3 −3x 57. x −e /3 + cos x 3x2 e−3x /9 − x4 sin x + 6x −e−3x /27 + 4x3 − cos x − 6 e−3x /81 − 12x2 − sin x + x3 e−3x dx 24x cos x − 24 sin x + x3 x2 2x 2 = − − − − e−3x + c 3 3 9 27 x4 cos xdx 62. = x4 sin x + 4x3 cos x − 12x2 sin x x2 − 24x cos x + 24 sin x + c ln x x3 /3 + −1 58. x x4 /12 + ex −x −2 x5 /60 + 4 x ex + The table will never terminate. 4x3 ex − 12x2 ex + 63. (a) Use the identity 24x ex − cos A cos B 1 24 ex + = [cos(A − B) + cos(A + B)] 2 x4 ex dx This identity gives π 4 3 2 = (x − 4x + 12x − 24x + 24)e + c x cos(mx) cos(nx)dx −π
  • 12. 6.2. INTEGRATION BY PARTS 371 π nπ 1 1 = [cos((m − n)x) = cos2 udu −π 2 n −nπ + cos((m + n)x)]dx nπ 1 1 1 1 sin((m − n)x) = u + cos(2u) =π = n 2 4 −nπ 2 m−n π π And then sin2 (nx)dx sin((m + n)x) −π + π m+n −π = (1 − cos2 (nx))dx =0 −π π π It is important that m = n because oth- = dx − cos2 (nx)dx erwise cos((m − n)x) = cos 0 = 1 −π −π (b) Use the identity = 2π − π = π sin A sin B 1 65. The only mistake is the misunderstanding of = [cos(A − B) − cos(A + B)] 2 antiderivatives. In this problem, ex e−x dx This identity gives π is understood as a group of antiderivatives of sin(mx) sin(nx)dx ex e−x , not a fixed function. So the subtraction −π π 1 by ex e−x dx on both sides of = [cos((m − n)x) −π 2 ex e−x dx = −1 + ex e−x dx − cos((m + n)x)]dx 1 sin((m − n)x) does not make sense. = π √ π 2 m−n π 66. V = π (x sin x)2 dx = π x2 sin xdx sin((m + n)x) 0 0 − m+n −π Using integration by parts twice we get =0 x2 sin xdx It is important that m = n because oth- erwise cos((m − n)x) = cos 0 = 1 = −x2 cos x + 2 x cos xdx 64. (a) Use the identity = −x2 cos x + 2(x sin x − sin xdx) cos A sin B = −x2 cos x + 2x sin x + 2 cos x + c 1 = [sin(B + A) − sin(B − A)] Hence, 2 π This identity gives V = (−x2 cos x + 2x sin x + 2 cos x) 0 π = π 2 − 4 ≈ 5.87 cos(mx) sin(nx) dx −π π 1 67. Let u = ln x, dv = ex dx = [sin((n + m)x) dx −π 2 du = , v = ex x − sin((n − m)x)] dx ex ex ln xdx = ex ln x − dx 1 cos((n + m)x) x = − ex 2 n+m ex ln xdx + dx = ex ln x + C π x cos((n − m)x) Hence, + n−m 1 −π ex ln x + dx = ex ln x + c =0 x (b) We have seen that 68. We can guess the formula: 1 1 cos2 xdx = x + cos(2x) + c ex (f (x) + f (x))dx = ex f (x) + c 2 4 Hence by letting u = nx: and prove it by taking the derivative: π cos2 (nx)dx d x (e f (x)) = ex f (x) + ex f (x) −π dx
  • 13. 372 CHAPTER 6. INTEGRATION TECHNIQUES b = ex (f (x) + f (x)) + f (x) (b − x) dx a 1 b 69. Consider, f (x)g (x) dx Consider x sin (b − x) dx 0 0 Choose u = g (x) and dv = f (x)dx, b b so that du = g (x) dx and , v = f (x) . = (b − x) sin xdx = (sin x) (b − x) dx 0 0 Hence, we have Now, consider 1 g (x)f (x)dx f (x) = x − sin x ⇒ f (x) = 1 − cos x 0 and f (x) = sin x. 1 1 = g (x) f (x)|0 − f (x)g (x) dx Therefore, using 0 f (b) = f (a) + f (a) (b − a) = (g (1) f (1) − g (0) f (0)) b 1 + f (x) (b − x) dx, − g (x)f (x) dx a 0 we get From the given data. b − sin b = 0 − sin 0 + f (0) (b − 0) 1 b = (0 − 0) − g (x)f (x) dx. + (sin x) (b − x) dx 0 0 Choose, u = g (x) and dv = f (x)dx, b so that,du = g (x) dx and v = f (x) . ⇒ |sin b − b| = x sin (b − x) dx . 0 Hence, we have 1 Further, − g (x)f (x) dx b b 0 |sin b − b| = x sin (b − x) dx ≤ xdx , 1 0 0 1 =− g (x) f (x)|0 − f (x) g (x) dx as sin (b − x) ≤ 1. 0 = − {(g (1) f (1) − g (0) f (0) ) b2 Thus, |sin b − b| ≤ . 1 2 − f (x) g (x) dx Therefore the error in the approximation 0 1 From the given data. sin x ≈ x is at most x2 . 1 2 = − (0 − 0 ) − f (x) g (x) dx 1 0 6.3 Trigonometric = f (x) g (x) dx. 0 Techniques of 70. Consider, Integration b b f (x) (b − x) dx = (b − x) f (x) dx 1. Let u = sin x, du = cos xdx a a Choose u = (b − x) and dv = f (x) dx, cos x sin4 xdx = u4 du so that du = −dx and v = f (x) . 1 5 1 = u + c = sin5 x + c Hence, we have: 5 5 b (b − x)f (x) dx 2. Let u = sin x, du = cos xdx a b cos3 x sin4 xdx = (1 − u2 )u4 du b = (b − x) f (x)|a + f (x) dx a u5 u7 b = − +c 5 7 = (0 − [(b − a) f (a)]) + f (x) dx sin x sin7 x 5 a b = − +c = − [(b − a) f (a)] + f (x)|a 5 7 = − [(b − a) f (a)] + f (b) − f (a) 3. Let u = sin 2x, du = 2 cos 2xdx. b π/4 f (x) (b − x) dx cos 2xsin3 2xdx a 0 = − [(b − a) f (a)] + f (b) − f (a) 1 1 1 1 u4 1 = u3 du = = f (b) = f (a) + (b − a) f (a) 2 0 2 4 0 8